celene0007
Thanks Received: 0
Vinny Gambini
Vinny Gambini
 
Posts: 8
Joined: November 13th, 2010
 
 
 

Q16 - computer operating system software

by celene0007 Mon Oct 17, 2011 9:33 pm

For this question, I was in between A and C. In the end I chose C because the business might have protected the system, but it does not mean it protected an individual computer. However, I still do not know why A should be eliminated.

Can someone shed some light on why A is an incorrect answer?
User avatar
 
maryadkins
Thanks Received: 640
Atticus Finch
Atticus Finch
 
Posts: 1261
Joined: March 23rd, 2011
 
 
 

Re: Q16 - computer operating system software

by maryadkins Wed Oct 19, 2011 8:00 pm

Isn't it fun to do a computer question from 1991? For one, because virus is written as "virus"?

(A) gives us a reverse logic problem. We're told that introducing variety into computer systems can virtually eliminate the unauthorized access/"virus" problem. That is:

Variety --> No Vandalism

(A) tells us if we don't have variety, we will have vandalism.

No Variety --> Vandalism

This is reverse logic. We have negated both sides without flipping.

Hope this helps clarify!
 
jardinsouslapluie5
Thanks Received: 0
Forum Guests
 
Posts: 59
Joined: April 22nd, 2012
 
 
trophy
Most Thankful
 

Re: Q16 - computer operating system software

by jardinsouslapluie5 Sun Jul 15, 2012 1:56 am

I am baffled about the last sentence in (C) but will not have protected every computer from viral invasion.

The stimulus says "unauthorized access to all the computers at the same time could be virtually eliminated." Sure, it is only the possibility, but how can we say that every computer will not be protected for certain?
 
KakaJaja
Thanks Received: 1
Forum Guests
 
Posts: 37
Joined: May 17th, 2012
 
 
 

Re: Q16 - computer operating system software

by KakaJaja Sun Jan 20, 2013 7:14 pm

I have the same question here. From where can we know C)? Thank you!
 
agersh144
Thanks Received: 6
Elle Woods
Elle Woods
 
Posts: 84
Joined: December 20th, 2012
 
 
 

Re: Q16 - computer operating system software

by agersh144 Thu Jul 18, 2013 11:47 am

KakaJaja Wrote:I have the same question here. From where can we know C)? Thank you!


I think on the modern tests to protect themselves from that very objection they would most likely have to put a "necessarily" before the "have protected" but again this is 1991 which must be kept in mind.
User avatar
 
ohthatpatrick
Thanks Received: 3805
Atticus Finch
Atticus Finch
 
Posts: 4661
Joined: April 01st, 2011
 
 
 

Re: Q16 - computer operating system software

by ohthatpatrick Wed Jul 31, 2013 11:07 pm

Here's a complete rundown of this one:

Question Type:
Inference

Reading Task:
Just a set of facts (even when it looks like an argument, as this one does). Don't look for a conclusion. Accept it all as true. If you see information that can be synthesized, do so.

What do we want out of the correct answer:
The safest, most provable idea we can find.

(A) This is a GUARANTEE that every business that doesn't add variety to its operating systems WILL lose data from a virus.
We can't justify such a sure thing. We aren't told that every business will be hacked at some point. We aren't told that adding variety is the ONLY way to secure data (so a business might not go the variety route but add a retinal scanner to each computer and protect data that way).

Structurally, what this answer is doing is an illegal negation of the conditional we're given:
Introduce variety --> eliminate risk of data loss
whereas (A) is saying
~ Introduce variety --> guarantee risk of data loss

(B) Again, the idea that ALL data will be destroyed is too extreme. The passage claims that the vandal "can destroy MUCH of the data", so we don't have justification for saying ALL.

(C) The conditional mentioned before says that if a business introduces variety, "unauthorized access to ALL computers at the same time could be virtually eliminated", so that's how we support "increased overall protection for its systems". The reason it says that this tactic "will not have protected every computer from viral invasion" is that this tactic does nothing to protect an individual computer from a virus. A vandal could still hop on my computer and give it a virus. What the "introducing variety" tactic does is prevent the vandal on my computer from "automatically gaining access to the data on all the computers". Basically, if every computer is a little different, then a vandal would have to put a virus on each computer, one at a time. I agree with an astute poster above that in a perfect world, this answer should say "will not necessarily protect every computer" or "will not, in doing so, have protected every computer". Still, this is the safest, MOST provable answer.

(D) This is too sweeping a statement. We're only talking about linked computer systems, so we have no ammunition for claiming that unlinked computers CANNOT be protected.

(E) The 'Fake' comparison of "easier access to data" doesn't match anything we were told.

(C) is the correct answer